Diễn Đàn MathScope

Diễn Đàn MathScope (http://forum.mathscope.org/index.php)
-   Các Bài Toán Đã Được Giải (http://forum.mathscope.org/forumdisplay.php?f=107)
-   -   Topic về bất đẳng thức (2) (http://forum.mathscope.org/showthread.php?t=21609)

Lan Phuog 10-07-2011 08:05 PM

Topic về bất đẳng thức (2)
 
Trích:

Cho $x,y,z>0,xyz=1 $. Cmr: $\sum \frac{x^3}{z^3}(\frac{y^2}{x^2}+\frac{z^2}{y^2})^2 \ge \sum (\frac{x^4}{y}+\frac{x}{y^4})+2\sum\frac{x^2}{y^2} $
bđt này có thể cm bằng việc sdụng Vornicu Schur,thông qua các bước đổi biến: $(x,y,z)-(\frac{a}{b}, \frac{b}{c}, \frac{c}{a}) $ và $(a^3,b^3,c^3)-(\frac{1}{a},\frac{1}{b},\frac{1}{c}) $

birain9x 10-07-2011 08:18 PM

Cho a,b,c>0.CMR
$\frac{a^2}{b}+\frac{b^2}{c}+\frac{c^2}{a}\geq 3\sqrt[4]{\frac{a^4+b^4+c^4}{3}} $

leviethai 10-07-2011 08:29 PM

Trích:

Nguyên văn bởi birain9x (Post 105283)
Cho a,b,c>0.CMR
$\frac{a^2}{b}+\frac{b^2}{c}+\frac{c^2}{a}\geq 3\sqrt[4]{\frac{a^4+b^4+c^4}{3}} $

Lời giải của mình ở đây, bạn tham khảo nhé :D.

[Only registered and activated users can see links. Click Here To Register...]

Bất đẳng thức mạnh hơn vẫn đúng nếu thay số $4 $ bởi số $5 $ hoặc $6 $ (tất nhiên cách chứng minh khó khăn hơn rất nhiều và khó có lời giải đẹp)

11112222 10-07-2011 09:03 PM

Bất đẳng thức hoán vị
 
Cho $a,b,c \ge 0 $. Chứng minh :
$\sum \frac{a}{b}\ge \frac{a^2+b^2+c^2}{ab+bc+ca}+2 $

Mệnh Thiên Tử 10-07-2011 09:06 PM

Ta có :

$\sum \frac{a}{b}=\sum \frac{a^2}{ab} $
Dùng bđt Cauchy - Schwarz :

$\sum \frac{a^2}{ab}\geq \frac{(\sum a)^2}{\sum ab} $
Nhưng :

$\frac{(\sum a)^2}{\sum ab}=\frac{\sum a^2}{\sum ab}+2 $
Được đpcm

Joe Dalton 10-07-2011 09:07 PM

Trích:

Nguyên văn bởi 11112222 (Post 105291)
Cho $a,b,c \ge 0 $. Chứng minh :
$\sum \frac{a}{b}\ge \frac{a^2+b^2+c^2}{ab+bc+ca}+2 $

Đề bài chính xác phải là $a,b,c>0 $.
Khi đó áp dụng Cauchy-Schwarz, ta có
$\sum \frac{a}{b} = \sum \frac{a^2}{ab} \ge \frac{(a+b+c)^2}{ab+bc+ca} = \frac{a^2+b^2+c^2}{ab+bc+ca}+2 $

MathForLife 10-07-2011 09:43 PM

Trích:

Nguyên văn bởi daitoancvp (Post 105265)
Chứng minh BĐT sau bằng 3 cách
(TST VN 2006)
. Cho $x,y,z $ thuộc đoạn $[1;2] $. Chứng minh rằng:
$(x + y + z)\left( {\dfrac{1}{x} + \dfrac{1}{y} + \dfrac{1}{z}} \right) \ge 6\left( {\dfrac{x}{{y + z}} + \dfrac{y}{{x + z}} + \dfrac{z}{{x + y}}} \right) $

Chào thầy, em xin trình bày cách giải bằng phương pháp dồn biến:
Do bất đẳng thức đã cho là thuần nhất nên khoảng $\left [ 1;2 \right ] $ có thể thay thế bởi khoảng $\left [ k;2k \right ] $ bất kì.
Không mất tính tổng quát, giả sử $c=max\left \{ a;b;c \right \} $ . Bất đẳng thức ở đề bài tương đương với:

$(a+b+c)\left ( \frac{6}{b+c}+\frac{6}{c+a}+\frac{6}{a+b}-\frac{1}{a}-\frac{1}{b}-\frac{1}{c} \right )\le 18 $

Chuẩn hoá $a+b+c=3 $, cần chứng minh:

$\frac{6}{b+c}+\frac{6}{c+a}+\frac{6}{a+b}-\frac{1}{a}-\frac{1}{b}-\frac{1}{c}\le 6 $

Đặt $2t=a+b $ và $f(a;b;c)=\frac{6}{b+c}+\frac{6}{c+a}+\frac{6}{a+b}-\frac{1}{a}-\frac{1}{b}-\frac{1}{c} $

Xét hiệu:

$f(a;b;c)-f(t;t;c)=\frac{6}{b+c}+\frac{6}{c+a}-\frac{24}{2c+a+b}-\left ( \frac{1}{a}+\frac{1}{b}-\frac{4}{a+b} \right ) $

$=(a-b)^2\left ( \frac{6}{(b+c)(c+a)(2c+a+b)}-\frac{1}{ab(a+b)} \right )\le 0 $

Thật vậy, vì cách chọn c nên:

$(a+c)(b+c)(2c+a+b)\ge 2a.2b.2(a+b)>6ab(a+b) $

Vậy chỉ cần chứng minh: $f(t;t;c)\le 6 $
Thay $c=3-2t $ và biểu thức, ta được dạng tương đương:

$\frac{(t-1)^2(4t-3)}{t(3-2t^2)(3-t)}\ge 0 $

Mà $2t\ge 2\ge c $ nên

$4t\ge 2t+c=3\Rightarrow 4t-3\ge 0 $


Kết hợp với trên ta có đpcm. Đẳng thức xảy ra khi $a=b=c $ hoặc $(a;b;c)=(k;k;2k) $.

hien123 10-07-2011 11:21 PM

Bài toán của anh Phạm Hữu Đức:
Nếu a, b, c là các số thưc dương thì ta có BĐT:
$\sum \sqrt{\frac{b+c}{a}}\geq \sqrt{6.\frac{a+b+c}{\sqrt[3]{abc}}}
$
@ Leviethai: Cho em hỏi nếu mũ 6 thì giải như thế, em được biết nó đang unsolved trên mathlinks

kid3494 11-07-2011 08:33 AM

Các bạn giúp mình bài tập này :
Cho $\left\{\begin{matrix}
x, y, z > 1 & \\
x + y + z = xyz &
\end{matrix}\right. $
Tìm min $\frac{x - 2}{y^{2}} + \frac{y - 2}{z^{2}} + \frac{z - 2}{x^{2}} $

birain9x 11-07-2011 09:50 AM

Trích:

Nguyên văn bởi kid3494 (Post 105337)
Các bạn giúp mình bài tập này :
Cho $\left\{\begin{matrix}
x, y, z > 1 & \\
x + y + z = xyz &
\end{matrix}\right. $
Tìm min $\frac{x - 2}{y^{2}} + \frac{y - 2}{z^{2}} + \frac{z - 2}{x^{2}} $

Đặt P là bt ở đề bài.Ta có $P=\sum (\frac{x-2}{y^2}+\frac{1}{y})-\sum (\frac{1}{y})=\sum (\frac{(x-1)+(y-1)}{y^2})-\sum \frac{x+y+z}{xyz} $
Suy ra $P=\sum (x-1)(\frac{1}{y^2}+\frac{1}{x^2})-\frac{x+y+z}{xyz}\geq \sum \frac{2(x-1)}{xy}-\frac{x+y+z}{xyz}=\frac{xy+yz+zx}{xyz}-2 $
Mà $(xy+yz+zx)^2\geq 3xyz(x+y+z)=3(xyz)^2 $ nên $\frac{xy+yz+zx}{xyz}\geq \sqrt 3 $.
Vậy $P\geq \sqrt3-2 $

asdfghj 11-07-2011 10:20 AM

Cho $abc=1 $
Tìm giá trị lớn nhất của
$M=\frac{a}{b^2+c^2+a}+\frac{b}{a^2+c^2+b}+\frac{c} {b^2+a^2+c} $

khtoan 11-07-2011 11:14 AM

Trích:

Nguyên văn bởi asdfghj (Post 105365)
Cho $abc=1 $
Tìm giá trị lớn nhất của
$M=\frac{a}{b^2+c^2+a}+\frac{b}{a^2+c^2+b}+\frac{c} {b^2+a^2+c} $

Viết bài toán dưới dạng :Cho $abc=1 $ và a,b,c dương và ta đi chứng minh $\sum \frac{a^3}{b^6+c^6+a^3}\leqslant 1
\Leftrightarrow \sum \frac{a^4bc}{b^6+c^6+a^4bc}\leqslant 1 $
Sử dụng bổ đề cơ bản $b^6+c^6\geqslant bc(b^4+c^4) $
Ta suy ra
$\sum \frac{a^4bc}{b^6+c^6+a^4bc}\leqslant \sum \frac{a^4bc}{bc(b^4+c^4)+a^4bc}=\sum \frac{a^4}{a^4+b^4+c^4}=1 $

Ta có đpcm.Đẳng thức xảy ra khi và chỉ khi $a=b=c=1 $

Nguyenhuyen_AG 11-07-2011 11:48 AM

Trích:

Nguyên văn bởi birain9x (Post 105355)
Đặt P là bt ở đề bài.Ta có $P
\ge\frac{xy+yz+zx}{xyz}-2 $

Ngoài ra bất đẳng thức này còn có thể chứng minh nhờ vào đánh giá sau đây

$xy^2z^2(y-z)^2\ge 0,\;\;yz^2x^2(z-x)^2\ge 0,\;\;zx^2y^2(x-y)^2\ge 0 $

Mệnh Thiên Tử 11-07-2011 12:45 PM

Cho$ a,b,c \in [1;3] $và a+b+c = 6
Chứng minh : $a^{3}+b^3+c^3\leq 36 $

daiduong1095 11-07-2011 01:25 PM

Trích:

Nguyên văn bởi Mệnh Thiên Tử (Post 105395)
Cho$ a,b,c \geq 0 $và a+b+c = 6
Chứng minh : $a^{3}+b^3+c^3\leq 36 $

Đề bài hay vậy.Cho $a=6 ,b=c=0 $ suy ra SAI!!X_X

Mệnh Thiên Tử 11-07-2011 02:17 PM

Xin lỗi , ghi nhầm điều kiện :
$a,b,c \in [1;3] $và a +b +c = 6
Chứng minh $a^3+b^3+c^3 \leq 36 $

birain9x 11-07-2011 02:53 PM

Đặt $a=x+1,b=y+1,c=z+1 $ thì $x,y,z $ thuộc $[0,2] $ và $x+y+z=3 $.
Ta cần cm $x^3+y^3+z^3+3(x^2+y^2+z^2)\leq 24 $
Giả sử $z=max(x,y,z) $ thì $z \geq 1 $
Ta có $x^3+y^3+z^3+3(x^2+y^2+z^2) \leq (x+y)^3+z^3+3(x+y)^2+3z^2=15z^2-45z+54=15(z-1)(z-2)+24 \leq 24 $

thaibinh 11-07-2011 03:08 PM

Một bài tương tự.
Cho $a,b,c\in \left [ 0;2 \right ];a+b+c=3.
CMR: a^{3}+b^{3}+c^{3}\leq 9 $

daiduong1095 11-07-2011 03:19 PM

Trích:

Nguyên văn bởi hien123 (Post 105317)
Bài toán của anh Phạm Hữu Đức:
Nếu a, b, c là các số thưc dương thì ta có BĐT:
$\sum \sqrt{\frac{b+c}{a}}\geq \sqrt{6.\frac{a+b+c}{\sqrt[3]{abc}}}
$

Mình làm thế này:
Chuẩn hóa $abc=1 $
BDT cần cm $\Leftrightarrow \sum\frac{b+c}{a}+2\sum\sqrt{\frac{(a+b)(a+c)}{bc} } \ge 6(a+b+c) $
Mà $2\sum\sqrt{\frac{(a+b)(a+c)}{bc}} \ge 2\sum \frac{a+\sqrt{bc}}{\sqrt{bc}}=2\sum a\sqrt{a}+6 $(C-S)
$=\sum(a\sqrt{a}+a\sqrt{a}+1)+3 \ge 3(a+b+c)+3 $

Do đó ta chỉ cần cm:
$\sum\frac{b+c}{a}+3 \ge 3(a+b+c) $ là xong!
Mà BDT này $\Leftrightarrow (a+b+c)(\frac{1}{a}+\frac{1}{b}+\frac{1}{c}) \ge 3(a+b+c) $
$\Leftrightarrow \frac{1}{a}+\frac{1}{b}+\frac{1}{c} \ge 3 $.Hiển nhiên theo BDT AM-GM
Suy ra đpcm.

symaoxinhxan 11-07-2011 03:24 PM

Hoàn toàn tương tự như bài của bạn birain9x;
Không mất tính tổng quát giả sử c max dẫn đến 1 $\leq $ c
$a^3+b^3+c^3\leqslant (a+b)^3+c^3=(3-c)^3+c^3\leq 9\Leftrightarrow (c-1)(c-2)\leq 0 $

thaibinh 11-07-2011 03:56 PM

Cho các số thực không âm a,b,c: $a+b+c=3;CMR:2\left ( a^{2}+b^{2}+c^{2} \right )+a^{2}b^{2}c^{2}\geq 7 $

khtoan 11-07-2011 04:14 PM

Trích:

Nguyên văn bởi thaibinh (Post 105430)
Cho các số thực không âm a,b,c: $a+b+c=3;CMR:2\left ( a^{2}+b^{2}+c^{2} \right )+a^{2}b^{2}c^{2}\geq 7 $

Đặt
$a+b+c=p ,ab+bc+ca=q ,abc=r $
Theo bất đẳng thức Cô si $q\leq 3 $

Theo Schur bậc 3 $r\geqslant \frac{4q-9}{3} $
Thay vào bất đẳng thức cần chứng minh ,ta được bất đẳng thức hiển nhiên đúng

$(16q-60)(q-3)\geqslant 0 $

Đẳng thức xảy ra khi và chỉ khi $a=b=c=1 $

symaoxinhxan 11-07-2011 04:20 PM

Đặt $a+b+c=p;ab+bc+ca=q;abc=r $
Theo bdt Schur ta có:$r\geqslant \frac{p(4q-p^2)}{9}=\frac{4q-9}{3}(p=3) $
Bdt cần chứng minh$\Leftrightarrow 2(3^2-2q)+\frac{(4q-9)^2}{9}\geq 7\Leftrightarrow (q-3,75)(q-3)\leqslant 0 $
Đúng theo Côsi:$q\leqslant 3 $

daiduong1095 11-07-2011 07:14 PM

Trích:

Nguyên văn bởi thaibinh (Post 105430)
Cho các số thực không âm a,b,c: $a+b+c=3;CMR:2\left ( a^{2}+b^{2}+c^{2} \right )+a^{2}b^{2}c^{2}\geq 7 $

Bài này thì có cách khác như sau:
BDT cần cm $\Leftrightarrow 2\left ( a^{2}+b^{2}+c^{2} \right )+a^{2}b^{2}c^{2}+2\geq 9=(a+b+c)^2 $
$\Leftrightarrow a^2+b^2+c^2+a^2b^2c^2+2 \ge 2(ab+bc+ca) $
Mà $a^2b^2c^2+1 \ge 2abc $
Do vậy ta chỉ cần cm:
$a^2+b^2+c^2+2abc+1 \ge 2(ab+bc+ca) $
BDT này quen quá rồi:D


Conan Edogawa 11-07-2011 09:52 PM

Bài này nhìn cũng đẹp :D

Cho $a,b,c>0 $. Cm $\frac{a}{b+c}+\frac{b}{c+a}+\frac{c}{a+b}\le \frac{3}{2}.\frac{{{a}^{2}}+{{b}^{2}}+{{c}^{2}}}{a b+bc+ca} $

Nguyenhuyen_AG 11-07-2011 10:10 PM

Trích:

Nguyên văn bởi Conan Edogawa (Post 105486)
Bài này nhìn cũng đẹp :D

Cho $a,b,c>0 $. Cm $\frac{a}{b+c}+\frac{b}{c+a}+\frac{c}{a+b}\le \frac{3}{2}.\frac{{{a}^{2}}+{{b}^{2}}+{{c}^{2}}}{a b+bc+ca} $

Chú ý rằng

$\frac{a(ab+bc+ca)}{b+c}=a^2+\frac{abc}{b+c} $

nên bất đẳng thức cần chứng minh tương đương với

$a^2+b^2+c^2+abc\left (\frac{1}{b+c} +\frac{1}{b+c} +\frac{1}{b+c} \right )\le\frac{3}{2}\left (a^2+b^2+c^2 \right ) $

hay là

$abc\left (\frac{1}{b+c} +\frac{1}{b+c} +\frac{1}{b+c} \right )\le\frac{1}{2}\left (a^2+b^2+c^2 \right ) $

Sử dụng đánh giá

$\frac{1}{a}+\frac{1}{b}\ge\frac{4}{a+b} $

ta có

$abc\left (\frac{1}{b+c} +\frac{1}{b+c} +\frac{1}{b+c} \right )\le \frac{abc}{2}.\left ( \frac{1}{a}+\frac{1}{b}+ \frac{1}{c} \right ) $

Từ đó đưa bài toán về chứng minh

$\frac{abc}{2}.\left ( \frac{1}{a}+\frac{1}{b}+ \frac{1}{c} \right )\le \frac{a^2+b^2+c^2}{2} $

hay là

$a^2+b^2+c^2\ge ab+bc+ca. $

Đây là một bất đẳng thức hiển nhiên đúng nên ta có điều phải chứng minh.

symaoxinhxan 11-07-2011 10:16 PM

Chuẩn hóa $a^2+b^2+c^2=3 $.Ta có:
BDT cần chứng minh
$\Leftrightarrow \sum \frac{a}{b+c}\leqslant \frac{9}{2(ab+bc+ca)}\Leftrightarrow \sum a^2+\sum \frac{abc}{b+c}\leqslant \frac{9}{2}\Leftrightarrow \sum \frac{abc}{b+c}\leq \frac{3}{2} $
Ta có:$b+c\geq 2\sqrt{bc}\Rightarrow\frac{abc}{b+c}\leq \frac{abc}{2\sqrt{bc}}=\frac{a\sqrt{bc}}{2}\leq a^2+bc\leq a^2+\frac{b^2+c^2}{2} $

DaiToan 11-07-2011 10:49 PM

Cho n số thực dương $\[
a_1 ,a_2 ,...a_n
\]
$thoả mãn $\[
a_1 + a_2 + ... + a_n = 2n \]
$. Tìm giá trị nhỏ nhất của $\[
P = \sqrt {a_1^3 + 1} + \sqrt {a_1^3 + 1} + ... + \sqrt {a_n^3 + 1}
\]
$

avip 11-07-2011 11:33 PM

Nhờ mọi người bài này:
Cho bộ số $(a_i) $ gồm $n $ số nguyên dương thỏa $\sum_{i=1}^n a_i = 100 $.
Tìm GTLN của $P = \sum_{i=1}^n \frac{a_i}{\sum_{k=i}^n a_k} $.

phaituankhan19 12-07-2011 09:51 AM

Cho ba số $$$a,b,c > 0$$ $ thỏa mãn $$$a + b + c = 1006$$ $. Chứng minh rằng

$$$\sqrt {2012a + {{{{\left( {b - c} \right)}^2}} \over 2}} + \sqrt {2012b + {{{{\left( {c - a} \right)}^2}} \over 2}} + \sqrt {2012c + {{{{\left( {a - b} \right)}^2}} \over 2}} \le 2012\sqrt 2 $$ $

Lil.Tee 12-07-2011 10:29 AM

Cho $a,b,c>0, a+b+c=3. $ Chứng minh:

$\frac{a+b}{ab+3}+\frac{b+c}{bc+3}+\frac{c+a}{ca+3} \ge \frac{3}{2} $

daiduong1095 12-07-2011 11:56 AM

Trích:

Nguyên văn bởi Lil.Tee (Post 105540)
Cho $a,b,c>0, a+b+c=3. $ Chứng minh:

$\frac{a+b}{ab+3}+\frac{b+c}{bc+3}+\frac{c+a}{ca+3} \ge \frac{3}{2} $

BDT cần cm tương đương với :
$4abc(ab+bc+ca)+6(a+b+c)(ab+bc+ca)+18abc+36(a+b+c) \ge 3(a^2b^2c^2+3abc(a+b+c)+9(ab+bc+ca)+27) $
$\Leftrightarrow 4qr+27 \ge 3r^2+9r+9q $

Ta có:$3r^2+9r+q(9-4r) \le3r^2+9r+3(9-4r)=3r(r-1)+27 \le 27 \Rightarrow $ đpcm.



khtoan 12-07-2011 12:28 PM

Trích:

Nguyên văn bởi daitoancvp (Post 105500)
Cho n số thực dương $\[
a_1 ,a_2 ,...a_n
\]
$thoả mãn $\[
a_1 + a_2 + ... + a_n = 2n \]
$. Tìm giá trị nhỏ nhất của $\[
P = \sqrt {a_1^3 + 1} + \sqrt {a_2^3 + 1} + ... + \sqrt {a_n^3 + 1}
\]
$

Áp dụng mincopski:
$
P = \sqrt {a_1^3 + 1} + \sqrt {a_2^3 + 1} + ... + \sqrt {a_n^3 + 1} \geq \sqrt{(\sum a^\frac{3}{2}_i)^2+n^2}\geq \sqrt{8n^2+n^2}=3n
$

Vậy $Min_P=3n $ khi và chỉ khi $a_i=2 $

Lan Phuog 12-07-2011 02:15 PM

Trích:

Nguyên văn bởi phaituankhan19 (Post 105537)
Cho ba số $$$a,b,c > 0$$ $ thỏa mãn $$$a + b + c = 6$$ $. Chứng minh rằng

$$$\sqrt {2012a + {{{{\left( {b - c} \right)}^2}} \over 2}} + \sqrt {2012b + {{{{\left( {c - a} \right)}^2}} \over 2}} + \sqrt {2012c + {{{{\left( {a - b} \right)}^2}} \over 2}} \le 2012\sqrt 2 $$ $

Hình như có nhầm lẫn rùi. $a+b+c=1006 $ mới đúng. Khi đó giả sử $a $ là số lớn nhất và tách đôi căn thức t1 ra,sau đó C-S cho 4 số,đồng bậc 2vế,bình phương lên sẽ ra được bđt hiển nhiên đúng. Dấu = xra khi 1số ~1006 và 2số còn lại ~0.

khtoan 12-07-2011 04:04 PM

Trích:

Nguyên văn bởi Conan Edogawa (Post 105486)
Bài này nhìn cũng đẹp :D

Cho $a,b,c>0 $. Cm $\frac{a}{b+c}+\frac{b}{c+a}+\frac{c}{a+b}\le \frac{3}{2}.\frac{{{a}^{2}}+{{b}^{2}}+{{c}^{2}}}{a b+bc+ca} $

Đóng góp 1 cách nữa: :D
Cộng 3 cho mỗi vế ,ta viết lại bài toán dưới dạng :
$
\Leftrightarrow \sum \frac{a+b+c}{b+c}\leqslant 3.\frac{a^2+b^2+c^2}{2ab+2bc+2ca}+3 $

Điều này $\Leftrightarrow (a+b+c)(\sum \frac{1}{a+b})\leqslant \frac{3(a+b+c)^2}{2ab+2bc+2ca} $
$
\Leftrightarrow \sum \frac{ab+bc+ca}{a+b}\leqslant \frac{3}{2}(a+b+c) $
$\Leftrightarrow \sum \frac{ab}{a+b}\leq \frac{(a+b+c)}{2} $ (Dễ chứng minh được bằng AM-GM)

Vậy ta có đpcm.Đẳng thức xảy ra $
\Leftrightarrow a=b=c $

phaituankhan19 12-07-2011 04:23 PM

Trích:

Nguyên văn bởi Lan Phuog (Post 105589)
Hình như có nhầm lẫn rùi. $a+b+c=1006 $ mới đúng. Khi đó giả sử $a $ là số lớn nhất và tách đôi căn thức t1 ra,sau đó C-S cho 4 số,đồng bậc 2vế,bình phương lên sẽ ra được bđt hiển nhiên đúng. Dấu = xra khi 1số ~1006 và 2số còn lại ~0.

Tôi xin lỗi, tôi đánh đề bị nhầm mất, cảm ơn bạn Lan Phuog nhé. Tôi sẽ sửa lại đề ngay đây.

king_math96 12-07-2011 04:38 PM

Trích:

Nguyên văn bởi khtoan (Post 105245)
Bài này được chế từ ý tưởng bài Polish MO 2005 có nội dung như sau:Cho a,b,c là các số dương.CMR:

$3\sqrt[9]{\frac{9a(a+b)}{2(a+b+c)^2}}+\sqrt[3]{\frac{6bc}{(a+b)(a+b+c)}}\leqslant 4 $

Đây là bài mình mới chế cũng theo ý tưởng trên :D
Cho a,b,c là các số dương.CMR:

$13\sqrt[13]{\frac{3b(a+2c)}{(a+b+c)(2a+b)}}+6\sqrt[6]{\frac{3a}{a+b+c}}\leqslant 19 $

Anh em thử sức nhá :D

cũng theo ý tưởng trên. Ta có:
$13.\sqrt[13]{\frac{3b(a+2c)}{(a+b+c)(2a+b)}} $$=13.\sqrt[13]{\frac{3b}{2b+a}.\frac{2b+a}{2a+b}.\frac{a+2c}{a+b +c}} $
$\leq \frac{3b}{2b+a}+ \frac{2b+a}{2a+b}+\frac{a+2c}{a+ b+c}+10. (1) $
Tương tự ta có: $6.\sqrt[6]{\frac{3a}{a+b+c}} \leq \frac{3a}{2a+b}+\frac{2a+b}{2b+a}+\frac{2b+a}{a+b+ c} +3 (2) $
Từ (1) và (2) ta cộng vế với vế suy ra ĐPCM. Dấu bằng khi và chỉ khi $a=b=c, $
------------------------------Cho 2n số thực dương $a_1,a_2,...a_n $ và $b_1,b_2,...b_n $ với n là số nguyên dương lớn hơn 1 thỏa mãn: $a_1.a_2...a_n=b_1.b_2...b_n $ và $b_1+b_2+...b_n=1 $;
$\sum{|a_i-a_j|} \leq \sum{|b_i-b_j|} $. với 1 $\leq i \leq j \leq n $
Tìm max của $A= a_1+a_2+...a_n. $
bài dưới là mình gõ sai, mod xóa giúp nhé. Thanks.

khtoan 12-07-2011 04:53 PM

Nice Solution king_math96:D,lời giải của bạn rất đúng và giống mình.1 bài nữa
Cho a,b,c dương.Chứng minh rằng :
$\frac{1}{3abc}+\frac{2}{a^3+b^3+c^3+3abc}\geqslant \frac{1}{a^2b+b^2c+c^2a}+\frac{1}{ab^2+bc^2+ca^2} $

Uy_Vũ 12-07-2011 04:55 PM

Trích:

Nguyên văn bởi phaituankhan19 (Post 105537)
Cho ba số $$$a,b,c > 0$$ $ thỏa mãn $$$a + b + c = 1006$$ $. Chứng minh rằng

$$$\sqrt {2012a + {{{{\left( {b - c} \right)}^2}} \over 2}} + \sqrt {2012b + {{{{\left( {c - a} \right)}^2}} \over 2}} + \sqrt {2012c + {{{{\left( {a - b} \right)}^2}} \over 2}} \le 2012\sqrt 2 $$ $

Chỉ cần chú ý:
$\sqrt{a(a+b+c)+\frac{(b-c)^2}{4}} \le a+\frac{b+c}{2} $

DaiToan 12-07-2011 05:15 PM

Trích:

Nguyên văn bởi daitoancvp (Post 105500)
Cho n số thực dương $\[
a_1 ,a_2 ,...a_n
\]
$thoả mãn $\[
a_1 + a_2 + ... + a_n = 2n \]
$. Tìm giá trị nhỏ nhất của $\[
P = \sqrt {a_1^3 + 1} + \sqrt {a_1^3 + 1} + ... + \sqrt {a_n^3 + 1}
\]
$

Thực ra mình tạo ra bài toán này là từ một trong hai bổ đề rat don gian sau:
Bổ đề 1: $\[
\sqrt {a^3 + 1} \ge \left| {2a - 1} \right|
\]
$
Bổ đề 2: $\[
a^3 + 1 \ge \frac{{(4a + 1)^3 }}{{81}}
\]
$


Múi giờ GMT. Hiện tại là 12:18 AM.

Powered by: vBulletin Copyright ©2000-2024, Jelsoft Enterprises Ltd.

[page compression: 46.08 k/49.61 k (7.12%)]